Determine the correct scientific notation form of the number. 627,000,000

Answers

Answer 1

Answer:

6.27 x 10 ^ 8

Step-by-step explanation:


Related Questions

HELP I WILL GUVE BRAINLIEST!

Answers

The answer is 32.527 or 32.

If you see a box on the top corner of the triangle, use just half of the triangle for this problem. Use it as a right triangle. The Pythagorean Theorem is a²+b²=c². [tex]\overline{CD}[/tex] is 23, so [tex]\overline{BD}[/tex] is also 23, because it's called a 45°-45°-90° triangle, due to the angles being the same. Type in √(23^2+23^2) into your calculator, or use an equation solver: 23²+23²=x², solve for x, and you will get 32.527 or 32.

solve for x 10=5/8x please

Answers

Answer:

x=16

Step-by-step explanation:

Answer:x=16

Step-by-step explanation: 10 divided by 5/8 is 16 so x equals 16

What is the quotient of (x3 - 3x2 + 5x – 3) = (x - 1)?
x2 - 2x - 3
x2 + 2x + 7
x2-3x + 8
x2 - 2x + 3

Answers

Note: Consider "÷" sign instead of "=".

Given:

[tex](x^3-3x^2+5x-3)\div (x-1)[/tex]

To find:

The quotient.

Solution:

We have,

[tex](x^3-3x^2+5x-3)\div (x-1)[/tex]

It can be written as

[tex]=\dfrac{x^3-3x^2+5x-3}{x-1}[/tex]

Splitting the middle terms, we get

[tex]=\dfrac{x^3-x^2-2x^2+2x+3x-3}{x-1}[/tex]

[tex]=\dfrac{x^2(x-1)-2x(x-1)+3(x-1)}{x-1}[/tex]

Taking out the common factor (x-1), we get

[tex]=\dfrac{(x-1)(x^2-2x+3)}{x-1}[/tex]

Cancel the common factors.

[tex]=x^2-2x+3[/tex]

The quotient is [tex]x^2-2x+3[/tex].

Therefore, the correct option is D.

Answer:

D

Step-by-step explanation:

EASY!!!! Answer correctly
I will give brainliest!!!

Answers

Answer:  37

Step-by-step explanation: i did it i think ¯\(°_o)/¯

If angle 1 and angle 2 are vertical angles then angle 1 and angle 2 are congruent

Answers

Answer:

[tex]\large\boxed{\textsf{True.}}[/tex]

Step-by-step explanation:

[tex]\textsf{This is a True or False question, where we are asked if vertical angles are equal.}[/tex]

[tex]\Large\underline{\textsf{What are Vertical Angles?}}[/tex]

[tex]\textsf{Vertical Angles are 2 angles that are opposite from each other, and are formed by}[/tex]

[tex]\textsf{the same, straight lines.}[/tex]

[tex]\textsf{Because they're formed by the same lines and are opposite from each other, vertical}[/tex]

[tex]\textsf{angles are congruent to each other.}[/tex]

[tex]\boxed{\mathtt{\angle 1 \cong \angle2}}[/tex]

whats the answer to this?

Answers

Answer: when a > 1, the graph is vertically stretched and steeper.

I hope this helps :)

The printed price of a DVD player is $30. If the buyer pays a sales tax of 8%, find the price at which the
DVD player is sold?

Answers

Answer:

It was sold at $32.40.

Step-by-step explanation:

hope this helpss :))

This graph shows transformations between
f(x) = 1/x and g(x)= a*(1/x). Use the drop-down menus to
identify the following functions.

Answers

Answer:

b

a

c

d

Step-by-step explanation:

I don’t know how to do it

Answers

Answer:

option D

Step-by-step explanation:

For the first 6% of Britney's salary, her employer matches 100% of her 401(k) contributions, and from 6% to 12%, Britney's employer matches 50% of her 401(k) contributions. Britney's salary is $30,000, and last year, she contributed $4000 to her 401(k) plan. What was her employer's contribution to the 401(k)?

A. $4000
B. $1800
C. $3600
D. $2700

Answers

Answer:

D. $2,700

Step-by-step explanation:

APE X

Write this linear equation in standard form: y−2=13(x−10)

A: x + 3y = 4

B: x - 3y = 4

C: x - 3y = 8

D: x - 3y = -4

Answers

Answer:

13x-y=128

Step-by-step explanation:

graph is shown below

Mary has been saving her weekly allowance for the past 20 weeks. She started with $100 and now has $300. Which equation can be used to find the amount of money Mary receives for her weekly allowance?

Answers

Answer:

Step-by-step explanation:

You should set this up as an y = mx + b equation. Since 300 is her final number, that will be your y value. She started with 100, so that will be your b value. 20 is the number related to the variable you don't know so that will be your m.

300 = 20x + 100

-100            -100

200 = 20x

divide both sides by 20

10 = x

NEED HELP PLEASE.............

Answers

C
E
(Maybe B, but not 100% sure though)
It’s C And E Have Ah Good Day


What is the third term of the sequence?
A
25
B
50
C. 35
D
20

Answers

Answer:

C

Step-by-step explanation:

35 is the third term of the sequence

Have a great day

8/34=?
can you show the work pls and thank you

Answers

The answer is 4/17 the work is below

At a dog breeder's property,

2
5
of the 60 dogs are black Labradors. This is

2
3
of the total black dogs.

How many black dogs are there in total?

Answers

45 black dogs because 2/3 of 60 is 45

Solve the Inequality

Answers

The answer would be b because if you add a -4 to a positive 4 you would get 0 same with -3 plus a positive 3 the symbol says it needs to be less or equal to 0

Palmer earns $84.00 for mowing 8 lawns. At this rate, how much will he earn if he mows 5
lawns?

Answers

Answer:

$52.50

Step-by-step explanation:

Divide to find the unit rate.

84 / 8 = $10.50 per lawn

Multiply.

10.5 * 5 = $52.50 for 5 lawns

Best of Luck!

Answer:

52.50

Step-by-step explanation:

a bicycle had a listed price of $607.98 before tax. if the sales tax rate is 8.25% find the cost of the bicycle with sales tax included. round your answer to the nearest cent if necessary​

Answers

Answer: $658.14

Step-by-step explanation:

The answer is $658.14 hope this helps:(

Charlotte wants to solve the equation 6(2x+1) – 3 = 4x – 5. Which step would be an appropriate first step for Charlotte to take to solve for x? Subtract 1 from both sides to combine like terms Add 4x to both sides to isolate the variable Distribute the 6 to eliminate the parentheses Combine 4x and -5 to simplify the problem

Answers

Answer:

Distribute the 6 to eliminate the parentheses

Mr. Gonzalez joined a gym that had an initial membership fee of $199 and a monthly cost of $35. How
many months would he have been a member if he paid a total of $829?

Answers

Answer:

sorry

Step-by-step explanation:

i need help answering these!! please help! 7-10

Answers

Answer:

there 7.6-z divided by 3

8.2k+s-2

9.8-s

10.5 times m divided by 2

Step-by-step explanation:

evaluate each of the following 25.6-4.4 solve​

Answers

21.2

The answer is 21.2 because of you subtract 4.4 from 25.6 it equals 21.2. My work is shown in picture.

ax2 + ___ + c = 0????? find​

Answers

Answer:

ax² + bx + c = 0

Step-by-step explanation:

ax² + bx + c = 0

its a Standard form of Quadratic Equation

-TheUnknownScientist

please please help me!!​

Answers

Answer:

Negative Association because it is going down left to right

i need some help please

Answers

the awnser will be 12 !!

have a good day !

Need help on this question

Answers

Answer:

How to calculate the amount of sales tax?

    Convert tax percentage into a decimal by moving the decimal point two spaces to the left.

    Multiple the pre-tax value by the newly calculated decimal value in order to find the cost of the sales tax.

    Add the sales tax value to the pre-tax value to calculate the total cost.

Step-by-step explanation:

Consider the two groups listed below. Which statement describes the sets?
. house
• complete mailing address

The relation (house, complete mailing address) is a function, but the relation (complete mailing address, house) is not.

The relation (complete mailing address, house) is a function, but the relation (house, complete mailing address) is not.

Both (house, complete mailing address) and (complete mailing address, house) are functions.

Neither the relation (house, complete mailing address) nor the relation (complete mailing address, house) is a function

Answers

Answer:

987

Step-by-step explanation:

yeah my teacer =

bruh can someone help me ill give brainlist

Answers

Answer:

x=2

Step-by-step explanation:

6+8x=6x+10

6+2x=10 Subtract 6 from both sides

2x=4 subtract 6 from both sides

x=2 divide by 2 on both sides

Hope this helps plz mark brainliest :D

List the first number greater than 5000 that is divisible by 2, 5, and 10?
( I can’t find one that’s 5 )

Answers

Answer:

5,010

Step-by-step explanation:

Can I have Brainliest? TYSMMMMMMMMM if it is divisible by 10 it is divisible by 5. divisible by 10 is usually even

Other Questions
Bias is favoring a view or perspective on an issue over another perspective. Question 1 options: True False What is the length of segment AC? A function is negative over the intervals{x 1-0 What are the zeros for the function?3-3,3-3 why do you hate careless people? please this is for an essay and i need help! it would help if you also explain why. thank you! In order to change a solid to a liquid or a liquid to a gas, what is needed?A high boiling pointAn increase in thermal energyA high melting pointA decrease in thermal energyHide hint for Question 9 jenet can make 4/5 on a necklace in 20 minutes.at this rate how many necklaces to the nearest tenth of a necklace can jenet make in 1 hour What is the answer to life? Chloe prepared 28 kilograms of dough after working 7 hours. How much dough did Chloe prepare if she worked for 8 hours? Assume the relationship is directly proportional. teddy Roosevelt espoused a very imperialistic foreign policy, and yet fought against monopolies at home. we're his foreign policy and domestic policy at the odds with each other? Which was a main reason for creating the Articles of Confederation? Systems by elimination: please help me..ASAP why are 6, 8, 10, 5, 12, 13 the pythagorean triples can someone help please!? I need help with these two questions Help with this math thing please! What did classical Greece contribute to modern Western civilization?A. abolition of slaveryB. women's suffrageC. monotheistic religionD. democratic government 10 (254) = Please Jenna has a piece of paper that is 99 3/4 square inch in Area.It is 10 1/2 inches long.What is the width of the piece of paper in inches? kal do i see you brother? plato banned certain songs because he don't approve of the instruments that were used